Side 1 av 1

romgeometri

Lagt inn: 21/05-2018 15:48
av mattenøtta
hei!

Når jeg skal finne det største volumet pyramiden kan få, finner jeg at x=3π /2. Når jeg da regner videre, får jeg feil svar for planet b og radiusen til kula, noe som betyr at jeg må ha funnet feil volum for pyramiden... Noen som ser hva jeg kan ha gjort feil? Jeg fant maks volum ved å finne toppunktet til V(t).

Re: romgeometri

Lagt inn: 21/05-2018 21:04
av DennisChristensen
mattenøtta skrev:hei!

Når jeg skal finne det største volumet pyramiden kan få, finner jeg at x=3π /2. Når jeg da regner videre, får jeg feil svar for planet b og radiusen til kula, noe som betyr at jeg må ha funnet feil volum for pyramiden... Noen som ser hva jeg kan ha gjort feil? Jeg fant maks volum ved å finne toppunktet til V(t).
$V(t)$ er en lineær funksjon i $t$, så vi ser med en gang at $V'(t) = 0$ er ekvivalent med likningen $\cos t = 0$, som har løsningene $t = \frac{\pi}{2}$ og $\frac{3\pi}{2}$ for $t\in [0,2\pi)$. Du kan putte disse verdiene for $t$ inn i uttrykket du fikk for $V(t)$ for å finne volumet.

Etter oppgave (b) står det at vi skal sette $t=0$ i resten av oppgaven, så $E = (2,2,-2)$. Oppgavene (c) og (d) angår ikke volumet av pyramiden, så du har ikke følgefeil, men heller en feil i utregningen din i oppgave (c) og/eller (d).

Re: romgeometri

Lagt inn: 22/05-2018 10:33
av mattenøtta
DennisChristensen skrev:
mattenøtta skrev:hei!

Når jeg skal finne det største volumet pyramiden kan få, finner jeg at x=3π /2. Når jeg da regner videre, får jeg feil svar for planet b og radiusen til kula, noe som betyr at jeg må ha funnet feil volum for pyramiden... Noen som ser hva jeg kan ha gjort feil? Jeg fant maks volum ved å finne toppunktet til V(t).
$V(t)$ er en lineær funksjon i $t$, så vi ser med en gang at $V'(t) = 0$ er ekvivalent med likningen $\cos t = 0$, som har løsningene $t = \frac{\pi}{2}$ og $\frac{3\pi}{2}$ for $t\in [0,2\pi)$. Du kan putte disse verdiene for $t$ inn i uttrykket du fikk for $V(t)$ for å finne volumet.

Etter oppgave (b) står det at vi skal sette $t=0$ i resten av oppgaven, så $E = (2,2,-2)$. Oppgavene (c) og (d) angår ikke volumet av pyramiden, så du har ikke følgefeil, men heller en feil i utregningen din i oppgave (c) og/eller (d).
Åja. Ser nå at jeg ikke leste oppgaven nøye nok. Brukte visst den t-en jeg fant i oppgava med volumet videre, i stedet for å sette t=0... Takk for svar!